CR-OG - 203 - In Physics journals ..particle accelerators..

This topic has expert replies
Junior | Next Rank: 30 Posts
Posts: 18
Joined: Fri May 23, 2008 6:45 am

Timer

00:00

Your Answer

A

B

C

D

E

Global Stats

I would appreciate if someone could explain my doubts on this question.
I have mentioned my doubt below in spoiler:

Doubt:
[spoiler]The OA is E.
But i dont understand what is wrong with B. If the wait time has declined than it means that decreased availibility of "particle aceelerators" is irrelevant. Meaning:
--- either lesser people are researching on this(an alternative explanation) --- or more equipments were added and therefore even if several were not in good condition, there still were several more that were good/usable. (another alternative explanation)[/spoiler]

Please suggest.


Journalist: In physics journals, the number of articles reporting the results of experiments involving particle accelerators was lower last year than it had been in previous years. Several of the particle accelerators at major research institutions were out of service the year before last for repairs, so it is likely that the low number of articles was due to the decline in availability of particle accelerators.
Which of the following, if true, most seriously undermines the journalist�s argument?
(A) Every article based on experiments with particle accelerators that was submitted for publication last year actually was published.
(B) The average time scientists must wait for access to a particle accelerator has declined over the last several years.
(C) The number of physics journals was the same last year as in previous years.
(D) Particle accelerators can be used for more than one group of experiments in any given year.
(E) Recent changes in the editorial policies of several physics journals have decreased the likelihood that articles concerning particle-accelerator research will be accepted for publication.

Junior | Next Rank: 30 Posts
Posts: 13
Joined: Fri Aug 15, 2008 9:41 am

by supershick » Sun Aug 17, 2008 12:16 pm

Timer

00:00

Your Answer

A

B

C

D

E

Global Stats

My answer:

Conclusion: low articles due to decline in availability of particle acclerators

Evidence: many PAs are being serviced

Need to weaken the conclusion by finding something that could cause low articles without being the decline in availability of PAs

I think best answer is (E) because if this were true, then PA articles would definitely be lower but it would negate the author's conclusion.

User avatar
Legendary Member
Posts: 543
Joined: Fri Jan 18, 2008 1:01 am
Thanked: 43 times
GMAT Score:580

Timer

00:00

Your Answer

A

B

C

D

E

Global Stats

The problem with statement (B) is that it negates the following fact made by author in his argument.

Several of the particle accelerators at major research institutions were out of service the year before last for repairs

In CR questions, you are not suppose to negate the facts. You need to weak the conclusion without touching the facts. So, in respect to do so, you need to find an alternate definition to do so, which certainly (E) do.

Read the following post and read the answer posted by Ron Purewal, for more understanding on such questions.

https://www.beatthegmat.com/consumers-in ... 13449.html

you can't question FACTS! you have to question LINES OF REASONING!! (as stated by Ron)

Junior | Next Rank: 30 Posts
Posts: 18
Joined: Fri May 23, 2008 6:45 am

by onesome » Sun Aug 17, 2008 1:18 pm

Timer

00:00

Your Answer

A

B

C

D

E

Global Stats

Thank you both supershick and codesnooker.

I still am not very clear.

Assumption that
===> "number of particle accelerators decreased and still the wait-time decreased"
can be inferred as
==> "the number of researchers on the topic decreased over the year "

Therefore B looks good to me in that case.

Added to the above the statement E begins with "Recent changes" - whereas the question is talking about last year and previous year


(E) Recent changes in the editorial policies of several physics journals have decreased the likelihood that articles concerning particle-accelerator research will be accepted for publication.

What do you think about that?

Appreciate your responses, thanks in advance.

User avatar
Legendary Member
Posts: 543
Joined: Fri Jan 18, 2008 1:01 am
Thanked: 43 times
GMAT Score:580

by codesnooker » Sun Aug 17, 2008 8:49 pm

Timer

00:00

Your Answer

A

B

C

D

E

Global Stats

This is a cause and effect type question. In such questions, you need to find the alternative cause that has actually turned out the effect.

So, if we look at the question,

Our facts (premises) are:-

1. In current year, the number of articles on PA is decreased.
2. The availability of PAs to RI were also decreased due to repairing work.

Conclusion:

1. The decrease in the number of articles was due to non-availability of PAs.

Now as I stated in my previous post, you are not suppose to challenge the facts. You need to find the answer that clearly overrides the conclusion without touching the facts given in the argument.

If you look at choice (B),

(B) The average time scientists must wait for access to a particle accelerator has declined over the last several years.


It states that average time of availability of PA to scientists has been decreased, which indirectly means that this year availability of PAs has been increased. It clearly challenge our stated fact (2), which is unacceptable in CR questions. Also according to it, the number of articles should be increased as PAs are easily available. This again challenges the fact (1). Hence, though no matter how promising this choices looks to you but it is certainly wrong. This is GMAT trap.

Now look at choice (E)
(E) Recent changes in the editorial policies of several physics journals have decreased the likelihood that articles concerning particle-accelerator research will be accepted for publication.


It give you an alternative cause of declining of the number of articles (fact 1) and it does not touches or negate any of the facts stated above.
Also we are talking about the number of articles published this year. Hence recent changes means the changes done this year that could affect the number of articles publication. So there is no timing problem with this choice.

Hope it clear to you now.

Junior | Next Rank: 30 Posts
Posts: 18
Joined: Fri May 23, 2008 6:45 am

by onesome » Mon Aug 18, 2008 4:45 am

Timer

00:00

Your Answer

A

B

C

D

E

Global Stats

thanks codesnooker .

I am on your side in regards to the rule => never challenge the fact stated - even if it is being challenged in one possible way by that choice..

Besides if i have to select from the remaining 4 choices than E is the only option.
But i would still say removing "recent" word from choice E would make it perfect.
But as they say in GMAT we are supposed to select the "best choice" - without expecting it to be perfectly correct. :)

I truly appreciate your time and thanks again :)

Junior | Next Rank: 30 Posts
Posts: 28
Joined: Thu Jun 24, 2010 11:12 am
Thanked: 7 times

by gnod » Thu Jun 09, 2011 7:33 pm

Timer

00:00

Your Answer

A

B

C

D

E

Global Stats

can an expert clarify why D wouldn't work in this case? I thought that if a PA gives access to more experiments any given time, it would weaken the author's claim that it will reduce the number of reports.

although... now that i'm looking at it, it kind of makes sense. if this were the case, and if we had a lot more reports from multiple experiments per machine... it makes sense that many more reports will be incomplete...

can someone share how to choose between D & E?

Thank you

Senior | Next Rank: 100 Posts
Posts: 30
Joined: Sat Aug 25, 2007 9:54 am
Thanked: 1 times

by what? » Thu Jul 14, 2011 12:25 am

Timer

00:00

Your Answer

A

B

C

D

E

Global Stats

So why is D wrong again... even the explanation given would suggest that D should be the answer.

D: If the accelerators can be used for multiple experiments, then it is reasonable to expect more
articles related to them, not fewer.

journalists argument depends on availability ..we just proved that availability is not an issue. Moreover, the term recent can mean anything. recent could mean in the last 5 years... in which case it could not be accountable for the decline in the number of articles last year.
Last edited by what? on Thu Jul 14, 2011 1:17 am, edited 1 time in total.

Legendary Member
Posts: 2330
Joined: Fri Jan 15, 2010 5:14 am
Thanked: 56 times
Followed by:26 members

by mundasingh123 » Thu Jul 14, 2011 1:06 am

Timer

00:00

Your Answer

A

B

C

D

E

Global Stats

Could someone give a valid reason why is d incorrect without citing E as a better Answer .
I Seek Explanations Not Answers

User avatar
Newbie | Next Rank: 10 Posts
Posts: 1
Joined: Sat Jul 03, 2010 12:29 pm

by josemauriciogg » Sun Oct 23, 2011 12:56 pm

Timer

00:00

Your Answer

A

B

C

D

E

Global Stats

Hi, I think that D is wrong because if it was correct, its a fact that doesn't permit the existence of the conclusion. Like, it's something that cancel the conclusion, but we have to weaken it, not cancel it.

Master | Next Rank: 500 Posts
Posts: 111
Joined: Mon Oct 03, 2011 12:08 pm

by nonameee » Wed Nov 16, 2011 1:57 am

Timer

00:00

Your Answer

A

B

C

D

E

Global Stats

Can I ask some of our experts to explain why D is incorrect? Thank you.

Master | Next Rank: 500 Posts
Posts: 111
Joined: Mon Oct 03, 2011 12:08 pm

by nonameee » Wed Nov 16, 2011 4:05 am

Timer

00:00

Your Answer

A

B

C

D

E

Global Stats

I also have had a hard time deciding between between (D) and (E).

(D) If particle accelerators can be used for more than one group of experiments in any given year, then the drop of the availability of accelerators due to their repairs could be offset by using available accelerators more.

(E) I dismissed this choice for two reasons:

1) The stimulus talks about articles reporting results of experiments involving particle accelerators; whereas (E) talks about articles concerning patricle-accelerator research, which IMO is not the same. And the reason I think these two are not the same is because particle accelerator research IMO studies accelerators, and experiments involving particle accelerators are concerned with behavior of particles in accelerators. E.g., (E) talks about CERN (European Laboratory for Particle Physics), stimulus talks about a research at one of CERN's accelerators.

2) Additionally, (E) talks about recent changes in editorial policies. The stimulus talks about events that are two years old.

Maybe I'm overthinking this one too much and maybe both underlined phrases are just the same. Maybe recent means two years ago. But anyway, I strongly think that (D) is a valid answer and not anywhere inferior to (E).

Here's an OG's explanation (BTW it's Q83 from OG12):

(D) If the accelerators can be used for multiple experiments, then it is reasonable to expect more articles related to them, not fewer.

Now, I don't know why we can infer that. Since some accelerators were out of service, then it is reasonable to assume that even though accelerators can be used to process more than one experiment, the workload on working accelerators would increase thereby not allowing the overall number of experiments to increase.

Any thoughts?

GMAT/MBA Expert

User avatar
GMAT Instructor
Posts: 16207
Joined: Mon Dec 08, 2008 6:26 pm
Location: Vancouver, BC
Thanked: 5254 times
Followed by:1268 members
GMAT Score:770

by Brent@GMATPrepNow » Wed Nov 16, 2011 7:05 am

Timer

00:00

Your Answer

A

B

C

D

E

Global Stats

onesome wrote: Journalist: In physics journals, the number of articles reporting the results of experiments involving particle accelerators was lower last year than it had been in previous years. Several of the particle accelerators at major research institutions were out of service the year before last for repairs, so it is likely that the low number of articles was due to the decline in availability of particle accelerators.
Which of the following, if true, most seriously undermines the journalist�s argument?
(A) Every article based on experiments with particle accelerators that was submitted for publication last year actually was published.
(B) The average time scientists must wait for access to a particle accelerator has declined over the last several years.
(C) The number of physics journals was the same last year as in previous years.
(D) Particle accelerators can be used for more than one group of experiments in any given year.
(E) Recent changes in the editorial policies of several physics journals have decreased the likelihood that articles concerning particle-accelerator research will be accepted for publication.
The conclusion here is a cause and effect conclusion. That is, the reduced availability of particle accelerators caused the low number of articles.


To weaken an X causes Y argument, we can show that:
- Y causes X
- Z causes Y
- X and Y are coincidental

Answer choice D doesn't accomplish any of the above. It suggests that there shouldn't have been a decline in publications in the first place (since more than 1 experiment can be conducted at an accelerator). I should also add that it if it were the case that each accelerator was already being used at maximum capacity (that is, no more experiments could be conducted at them), then the decreased availability would, indeed, cause a drop in publications. So D does not necessarily hurt the argument.

Answer choice E, however, suggests that something else (i.e., Z causes Y) caused the decline in publications.

Cheers,
Brent
Brent Hanneson - Creator of GMATPrepNow.com
Image

Master | Next Rank: 500 Posts
Posts: 111
Joined: Mon Oct 03, 2011 12:08 pm

by nonameee » Wed Nov 16, 2011 12:26 pm

Timer

00:00

Your Answer

A

B

C

D

E

Global Stats

Brent, thanks a lot for your reply.

I got it.

The effect: There were fewer articles reporting the results of experiments involving particle accelerators.

The stated cause (the one we should attack): Some accelerators were unavailable for research.

To refute the cause and effect relationship we need to, for example, find an alternative cause.

(D) doesn't accomplish this because had it been true that other functioning accelerators were available for multiple experiments, the effect would have never occurred in the first place. Well of course if those accelerators hadn't been already used at their full capacities. But even if that were the case, (D) still wouldn't be a great alternative cause of the decline in the number of published articles.

(E) provides an alternative explanation of the stated effect.

The only thing that still bothers me is the word 'lately'. Could you please comment on it?

Besides this one minor point, everything's clear now.


Thanks a lot.

GMAT/MBA Expert

User avatar
GMAT Instructor
Posts: 16207
Joined: Mon Dec 08, 2008 6:26 pm
Location: Vancouver, BC
Thanked: 5254 times
Followed by:1268 members
GMAT Score:770

by Brent@GMATPrepNow » Wed Nov 16, 2011 12:37 pm

Timer

00:00

Your Answer

A

B

C

D

E

Global Stats

nonameee wrote: The only thing that still bothers me is the word 'lately'. Could you please comment on it?
Yeah, it would have been better had the changes occurred a year ago. "Recently" could mean last week. Or it could mean 1 year a ago.

However, remember that our goal is not to destroy the argument. All we need to do is weaken it.
Answer choice E is the only one that puts a dent (however slight) in the conclusion by suggesting that it MAY be Z that causes Y (and not X)

Cheers,
Brent
Last edited by Brent@GMATPrepNow on Fri Nov 15, 2013 7:39 am, edited 1 time in total.
Brent Hanneson - Creator of GMATPrepNow.com
Image